Why Must the Hessian Matrix Be Symmetric at a Critical Point?

Click For Summary
SUMMARY

The Hessian matrix of a function f: R^2 -> R at a critical point must be symmetric due to the equality of mixed partial derivatives, as established by Clairaut's theorem. In the provided example, the matrix A = [[1, -2], [2, 3]] is not symmetric because the off-diagonal elements are not equal (fxy ≠ fyx). This violation confirms that the Hessian cannot represent the second derivatives of a function that is class C^3, where the third derivatives exist and are continuous.

PREREQUISITES
  • Understanding of multivariable calculus, specifically critical points.
  • Familiarity with the properties of the Hessian matrix.
  • Knowledge of Clairaut's theorem regarding mixed partial derivatives.
  • Concept of functions classified as C^3, including continuity of derivatives.
NEXT STEPS
  • Study the properties of the Hessian matrix in detail.
  • Learn about Clairaut's theorem and its implications for mixed partial derivatives.
  • Explore examples of C^3 functions and their critical points.
  • Investigate the application of the Hessian matrix in optimization problems.
USEFUL FOR

Students and professionals in mathematics, particularly those studying calculus, optimization, and differential equations, will benefit from this discussion.

AndreTheGiant
Messages
26
Reaction score
0

Homework Statement



Given a function f: R^2 -> R of class C^3 with a critical point c.

Why CANNOT the hessian matrix of f at point c be given by:

1 -2
2 3


Homework Equations





The Attempt at a Solution



So first i want to clarify this.

When it says f: R^2 -> R, that means the function is of two variables (x and y)?

And when it says class C^3 that means the third derivative of the function exists and is continuous. So would a function be x^3 or x^4? the third derivative would be 24x for x^4 and is continuous. The third derivative of x^3 would be 6.

I'm not sure about the answer..
 
Physics news on Phys.org
AndreTheGiant said:

Homework Statement



Given a function f: R^2 -> R of class C^3 with a critical point c.

Why CANNOT the hessian matrix of f at point c be given by:

1 -2
2 3


Homework Equations





The Attempt at a Solution



So first i want to clarify this.

When it says f: R^2 -> R, that means the function is of two variables (x and y)?

And when it says class C^3 that means the third derivative of the function exists and is continuous. So would a function be x^3 or x^4? the third derivative would be 24x for x^4 and is continuous. The third derivative of x^3 would be 6.

I'm not sure about the answer..

IF your matrix A above was a Hessian, what would the number a(2,2) = -2 represent? What would the number a(2,1) = +2 represent?

RGV
 
Ah ok. I think i got it. In the hessian which is given by

fxx fxy

fyx fyy

fxy is not equal to fyx which should be the case for mixed partials?
 
AndreTheGiant said:
Ah ok. I think i got it. In the hessian which is given by

fxx fxy

fyx fyy

fxy is not equal to fyx which should be the case for mixed partials?

Yes, exactly.

RGV
 
I have a question considering the applicability of Hessian matrix.

So, Can I use Hessian to prove that x^y > y^x whenever y > x >= e.

At first I start by multiplying by ln() => y*ln(x) > x*ln(y)

Is it enough, if I take g(x,y) such that g(x,y) = y*ln(x) - x*ln(y) > 0 and show det(H(g)) < 0 whenever y > x >= e?

My purpose with this is to show that there are no real local or global critical points in g(x,y) when y > x >= e, and conclude that x^y - y^x diverges. I am not sure if I can use Hessian to draw that kind of conclusion.
 
Hi Viliperi, welcome to PF, please start a new thread if you have a question as opposed to resurrecting an old one. You're more likely to get an anser that way as well - thanks
 
Question: A clock's minute hand has length 4 and its hour hand has length 3. What is the distance between the tips at the moment when it is increasing most rapidly?(Putnam Exam Question) Answer: Making assumption that both the hands moves at constant angular velocities, the answer is ## \sqrt{7} .## But don't you think this assumption is somewhat doubtful and wrong?

Similar threads

Replies
4
Views
2K
  • · Replies 3 ·
Replies
3
Views
2K
  • · Replies 8 ·
Replies
8
Views
2K
  • · Replies 8 ·
Replies
8
Views
4K
  • · Replies 7 ·
Replies
7
Views
3K
  • · Replies 4 ·
Replies
4
Views
1K
  • · Replies 24 ·
Replies
24
Views
4K
  • · Replies 3 ·
Replies
3
Views
1K
  • · Replies 8 ·
Replies
8
Views
7K
Replies
1
Views
2K